Discrete Math implications by rules of inference

Click For Summary
The discussion centers on the validity of using premises in a proof involving rules of inference. It clarifies that premises, such as p, remain valid throughout the proof and are not "used up" like statements in a conditional proof. Participants confirm that both p and q can be utilized at any point in the proof since they are established premises. The explanation emphasizes the distinction between premises and hypotheses in logical proofs. Understanding this concept is crucial for correctly applying rules of inference in discrete mathematics.
r0bHadz
Messages
194
Reaction score
17

Homework Statement


p→(q→r)
¬q →¬p
p
-----------------------
∴r

Homework Equations

The Attempt at a Solution


My book gives the following solution:

(1) p - premise
(2) ¬q→¬p premise
(3) q, (1) and (2) and rule of detachment,
(4) p and q, law of conjuctive addition
.
.
.

Can anyone explain to me why you can use p on step 4?
Since in step 3, you are using step 1 and step 2, and p comes from step 1

Does the p not get "used" up? I don't understand why you're able to use it again.
 
Physics news on Phys.org
No the p does not get used up. Since it is a given assumption rather than a hypothesis used to open a conditional proof, it is valid throughout the proof. The only case where statements get 'used up' is where they are made in a conditional proof, in which case they are not valid outside the conditional proof. Since there are no conditional proofs used in what you wrote above, that does not happen here.
 
  • Like
Likes r0bHadz
andrewkirk said:
No the p does not get used up. Since it is a given assumption rather than a hypothesis used to open a conditional proof, it is valid throughout the proof. The only case where statements get 'used up' is where they are made in a conditional proof, in which case they are not valid outside the conditional proof. Since there are no conditional proofs used in what you wrote above, that does not happen here.

Hmm great explanation.

Am I right when i say, so because P is the premise, I can use that whenever I want because its never going to not be the premise?

Similarly, because I deducted q in step 3, I can always use q just like I can always use p?
 
r0bHadz said:
Hmm great explanation.

Am I right when i say, so because P is the premise, I can use that whenever I want because its never going to not be the premise?

Similarly, because I deducted q in step 3, I can always use q just like I can always use p?
Yes. Both of those are valid.
 
  • Like
Likes r0bHadz
Question: A clock's minute hand has length 4 and its hour hand has length 3. What is the distance between the tips at the moment when it is increasing most rapidly?(Putnam Exam Question) Answer: Making assumption that both the hands moves at constant angular velocities, the answer is ## \sqrt{7} .## But don't you think this assumption is somewhat doubtful and wrong?

Similar threads

  • · Replies 24 ·
Replies
24
Views
4K
  • · Replies 1 ·
Replies
1
Views
1K
  • · Replies 16 ·
Replies
16
Views
2K
  • · Replies 3 ·
Replies
3
Views
1K
  • · Replies 4 ·
Replies
4
Views
1K
  • · Replies 5 ·
Replies
5
Views
1K
  • · Replies 2 ·
Replies
2
Views
2K
Replies
7
Views
2K
  • · Replies 8 ·
Replies
8
Views
2K
  • · Replies 2 ·
Replies
2
Views
2K